Probability of normal distribution

Click For Summary
The discussion revolves around finding the normal approximation for the binomial probability P(x = 4, 5) with n=14 and p=0.5. The calculations yield a mean (μ) of 7 and a standard deviation (σ) of approximately 1.87, leading to a z-score of -1.61. The participant's computed probability differs from the book's answer of 0.1812, raising concerns about the accuracy of the normal approximation in this case. It is noted that the approximation is not reliable due to the relatively small sample size and the z-score being significantly below the mean. A larger sample size or a z-score closer to the mean would improve the approximation's accuracy.
g.lemaitre
Messages
267
Reaction score
2

Homework Statement



find the normal approximation for the binomial probability P(x = 4,5) where n=14 and p = .5.

Homework Equations



μ = np
σ = sqrt(npq)

z = (x - μ)/σ

The Attempt at a Solution



p = .5 q = .5

μ = 14*.5 = 7

σ = sqrt(14 * .5 * .5) = 1.87

z = (4 - 7)/1.87 = -1.61

(my book uses tables to convert the z score into the probability of getting x < 4)

z = -1.61 = .5 - .4463 = .0537

The book says the answer is .1812 which is not what I'm getting.


z =
 
Physics news on Phys.org
it's rare that such a simple problem takes this long. let me provide the example from the book

Screenshot2012-10-06at63129PM.png


Screenshot2012-10-06at63132PM.png
 
g.lemaitre said:
it's rare that such a simple problem takes this long. let me provide the example from the book

Screenshot2012-10-06at63129PM.png


Screenshot2012-10-06at63132PM.png

The normal approximation is not very good in this example (because N = 25 is not very large and z is more than 2 standard deviations below the mean). P_exact = 0.0148904, while the continuity-corrected normal approximation is about 0.020152 (so using the normal gives about a 35% error). The normal approximation would be better if N were larger or z were closer to the mean.

RGV
 
Question: A clock's minute hand has length 4 and its hour hand has length 3. What is the distance between the tips at the moment when it is increasing most rapidly?(Putnam Exam Question) Answer: Making assumption that both the hands moves at constant angular velocities, the answer is ## \sqrt{7} .## But don't you think this assumption is somewhat doubtful and wrong?

Similar threads

  • · Replies 4 ·
Replies
4
Views
2K
Replies
2
Views
2K
  • · Replies 3 ·
Replies
3
Views
2K
  • · Replies 3 ·
Replies
3
Views
2K
  • · Replies 2 ·
Replies
2
Views
1K
  • · Replies 8 ·
Replies
8
Views
4K
  • · Replies 1 ·
Replies
1
Views
2K
  • · Replies 6 ·
Replies
6
Views
3K
Replies
1
Views
1K
Replies
1
Views
2K